Nurs 10 Study Guide Exam 2

अब Quizwiz के साथ अपने होमवर्क और परीक्षाओं को एस करें!

A nurse working in a community health clinic writes nursing diagnoses for patients and their families. Which nursing diagnoses are correctly written as three-part nursing diagnoses?

1) Disabled Family Coping related to lack of knowledge about home care of child on ventilator 2) Imbalanced Nutrition: Less Than Body Requirements related to inadequate caloric intake while striving to excel in gymnastics as evidenced by 20-lb weight loss since beginning the gymnastic program, and greatly less than ideal body weight when compared to standard height-weight charts 3) Need to learn how to care for child on ventilator at home related to unexpected discharge of daughter after 3-month hospital stay as evidenced by repeated comments "I cannot do this," "I know I'll harm her because I'm not a nurse," and "I can't do medical things" 4) Spiritual Distress related to inability to accept diagnosis of terminal illness as evidenced by multiple comments such as "How could God do this to me?" "I don't deserve this," "I don't understand. I've tried to live my life well," and "How could God make me suffer this way?" 5) Caregiver Role Strain related to failure of home health aides to appropriately diagnose needs of family caregivers and initiate a plan to facilitate coping as evidenced by caregiver's loss of weight and clinical depression a) (1) and (3) b) (2) and (4) c) (1), (2), and (3) d) (1), (2), (3), (4), and (5) Ans: b. (1) is a two-part diagnosis, (3) is written in terms of needs and not an unhealthy response, and (5) is a legally inadvisable statement which blames home health aides for the patient's problem. Statements that may be interpreted as libel or that imply nursing negligence are legally hazardous to all the nurses caring for the patient. Assigning blame in the written record is problematic.

A nurse is writing nursing diagnoses for patients in a psychiatrist's office. Which nursing diagnoses are correctly written as two-part nursing diagnoses?

1) Ineffective Coping related to inability to maintain marriage 2) Defensive Coping related to loss of job and economic security 3) Altered Thought Processes related to panic state 4) Decisional Conflict related to placement of parent in a long-term care facility a) (1) and (2) b) (3) and (4) c) (1), (2), and (3) d) (1), (2), (3), and (4) Ans: d. Each of the four diagnoses is a correctly written two-part diagnostic statement that includes the problem or diagnostic label and the etiology or cause.

Read the following scenario and identify the adjective used to describe the characteristics of patient data that are numbered below. Place your answers on the lines provided. The nurse is conducting an initial assessment of a 79-year-old female patient admitted to the hospital with a diagnosis of dehydration. The nurse (1) uses clinical reasoning to identify the need to perform a comprehensive assessment and gather the appropriate patient data. (2) First the nurse asks the patient about the most important details leading up to her diagnosis. Then the nurse (3) collects as much information as possible to understand the patient's health problems; (4) collects the patient data in an organized manner; (5) verifies that the data obtained is pertinent to the patient care plan; and (6) records the data according to facility's policy. (1) ___________________ (2) ___________________ (3) ___________________ (4) ___________________ (5) ___________________ (6) ___________________

Ans: (1) Purposeful: The nurse identifies the purpose of the nursing assessment (comprehensive) and gathers the appropriate data. (2) Prioritized: The nurse gets the most important information first. (3) Complete: The nurse gathers as much data as possible to understand the patient health problem and develop a care plan. (4) Systematic: The nurse gathers the information in an organized manner. (5) Accurate and relevant: The nurse verifies that the information is reliable. (6) Recorded in a standard format: The nurse records the data according to the facility's policy so that all caregivers can easily access what is learned.

Read the following patient scenario and identify the step of the nursing process represented by each numbered and boldfaced nursing activity. Annie seeks the help of the nurse in the student health clinic because she suspects that her roommate, Angela, suffered date rape. She is concerned because Angela chose not to report the rape and does not seem to be coping well. (1) After talking with Annie, the nurse learns that although Angela blurted out that she had been raped when she first came home, since then she has refused verbalization about the rape ("I don't want to think or talk about it"), has stopped attending all college social activities (a marked change in behavior), and seems to be having nightmares. After analyzing the data, the nurse believes that Angela might be experiencing (2) rape-trauma syndrome: silent reaction. Fortunately, Angela trusts Annie and is willing to come to the student health center for help. A conversation with Angela confirms the nurse's suspicions, and problem identification begins. The nurse talks further with Angela (3) to develop some treatment goals and formulate outcomes. The nurse also begins to think about the types of nursing interventions most likely to yield the desired outcomes. In the initial meeting with Angela, (4) the nurse encourages her expression of feelings and helps her to identify personal coping strategies and strengths. The nurse and Angela decide to meet in 1 week (5) to assess her progress toward achieving targeted outcomes. If she is not making progress, the care plan might need to be modified. (1) _____________________ (2) _____________________ (3) _____________________ (4) _____________________ (5) _____________________

Ans: (1) is an illustration of assessing: the collection of patient data. (2) is an illustration of the identification of a nursing diagnosis: a health problem that independent nursing intervention can resolve. (3) is an illustration of planning: outcome identification and related nursing interventions. (4) is an illustration of implementing: carrying out the care plan. (5) is an illustration of evaluating: measuring the extent to which Angela has achieved targeted outcomes.

A nurse is interviewing a newly admitted patient. Which question is considered culturally sensitive?

a) "Do you think you will be able to eat the food we have here?" b) "Do you understand that we can't prepare special meals?" c) "What types of food do you eat for meals?" d) "Why can't you just eat our food while you are here?" Ans: c. Asking patients what types of foods they eat for meals is culturally sensitive. The other questions are culturally insensitive.

A nurse is assessing a patient for tactile disturbances. Which question asked by the nurse would be appropriate for this assessment?

a) "Have you been experiencing any strange tastes lately?" b) "Have you smelled odors lately that other cannot smell?" c) "Can you tell me what I am placing in your hand right now?" d) "Have you found it difficult to communicate verbally?" Ans: c. When the nurse asks: "Can you tell me what I am placing in your hand right now?" the nurse is assessing for tactile disturbances. When the nurse asks: "Have you been experiencing any strange tastes lately?" the nurse is assessing for gustatory disturbances. The question: "Have you smelled odors lately that others cannot smell?" assesses for olfactory disturbances. The question: "Have you found it difficult to communicate verbally?" assesses for transmission-perception-reaction.

A nurse is using the ISBARR physician reporting system to report the deteriorating mental status of Mr. Sanchez, a patient who has been prescribed morphine via a patient-controlled analgesia pump (PCA) for pain related to pancreatic cancer. Place the following nursing statements related to this call in the correct ISBARR order.

a) "I am calling about Mr. Sanchez in Room 202 who is receiving morphine via a PCA pump for pancreatic cancer." b) "Mr. Sanchez has been difficult to arouse and his mental status has changed over the past 12 hours since using the pump." c) "You want me to discontinue the PCA pump until you see him tonight at patient rounds." d) "I am Rosa Clark, an RN working on the second floor of South Street Hospital." e) "Mr. Sanchez was admitted 2 days ago following a diagnosis of pancreatic cancer." f) "I think the dosage of morphine in Mr. Sanchez's PCA pump needs to be lowered." Ans: d, a, e, b, f, c. The order for ISBARR is: Identity/Introduction, Situation, Background, Assessment, Recommendation, and Read-back.

A nurse is discharging a patient from the hospital following a heart stent procedure. The patient asks to see and copy his medical record. What is the nurse's best response?

a) "I'm sorry, but patients are not allowed to copy their medical records." b) "I can make a copy of your record for you right now." c) "You can read your record while you are still a patient, but copying records is not permitted according to HIPAA rules." d) "I will need to check with our records department to get you a copy." Ans: d. According to HIPAA, patients have a right to see and copy their health record; update their health record; get a list of the disclosures a health care institution has made independent of disclosures made for the purposes of treatment, payment, and health care operations; request a restriction on certain uses or disclosures; and choose how to receive health information. The nurse should be aware of facility policies regarding the patient's right to access and copy records.

A nurse is writing an evaluative statement for a patient who is trying to lower cholesterol through diet and exercise. Which evaluative statement is written correctly?

a) "Outcome not met." b) "1/21/20—Patient reports no change in diet." c) "Outcome not met. Patient reports no change in diet or activity level." d) "1/21/20—Outcome not met. Patient reports no change in diet or activity level." Ans: d. The evaluative statement must contain a date; the words "outcome met," "outcome partially met," or "outcome not met"; and the patient data or behaviors that support this decision. The other answer choices are incomplete statements.

The nurse is admitting a 35-year-old pregnant woman to the hospital for treatment of preeclampsia. The patient asks the nurse: "Why are you doing a history and physical exam when the doctor just did one?" Which statements best explain the primary reasons a nursing assessment is performed? Select all that apply.

a) "The nursing assessment will allow us to plan and deliver individualized, holistic nursing care that draws on your strengths." b) "It's hospital policy. I know it must be tiresome, but I will try to make this quick!" c) "I'm a student nurse and need to develop the skill of assessing your health status and need for nursing care." d) "We want to make sure that your responses to the medical exam are consistent and that all our data are accurate." e) "We need to check your health status and see what kind of nursing care you may need." f) "We need to see if you require a referral to a physician or other health care professional." Ans: a, e, f. Medical assessments target data pointing to pathologic conditions, whereas nursing assessments focus on the patient's responses to health problems. The initial comprehensive nursing assessment results in baseline data that enable the nurse to make a judgment about a patient's health status, the ability to manage his or her own health care and the need for nursing. It also helps nurses plan and deliver individualized, holistic nursing care that draws on the patient's strengths and promotes optimum functioning, independence, and well-being, and enables the nurse to refer the patient to a physician or other health care professional, if indicated. The fact that this is hospital policy is a secondary reason, and although it may be true that a nurse may need to develop assessment skills, it is not the chief reason the nurse performs a nursing history and exam. The assessment is not performed to check the accuracy of the medical examination.

A student nurse attempts to perform a nursing history for the first time. The student nurse asks the instructor how anyone ever learns all the questions the nurse must ask to get good baseline data. What would be the instructor's best reply?

a) "There's a lot to learn at first, but once it becomes part of you, you just keep asking the same questions over and over in each situation until you can do it in your sleep!" b) "You make the basic questions a part of you and then learn to modify them for each unique situation, asking yourself how much you need to know to plan good care." c) "No one ever really learns how to do this well because each history is different! I often feel like I'm starting afresh with each new patient." d) "Don't worry about learning all of the questions to ask. Every facility has its own assessment form you must use." Ans: b. Once a nurse learns what constitutes the minimum data set, it can be adapted to any patient situation. It is not true that each assessment is the same even when using the same minimum data set, nor is it true that each assessment is uniquely different. Nurses committed to thoughtful, person-centered practice tailor their questions to the uniqueness of each patient and situation. Answer d is incorrect because relying solely on standard facility assessment tools does not allow for individualized patient care or critical thinking.

When the initial nursing assessment revealed that a patient had not had a bowel movement for 2 days, the student nurse wrote the diagnostic label "constipation." What would be the instructor's BEST response to this student's diagnosis?

a) "Was this diagnosis derived from a cluster of significant data or a single clue?" b) "This early diagnosis will help us manage the problem before it becomes more acute." c) "Have you determined if this is an actual or a possible diagnosis?" d) "This condition is a medical problem that should not have a nursing diagnosis." Ans: a. Nursing diagnoses should always be derived from clusters of significant data rather than from a single cue. A data cluster is a grouping of patient data or cues that point to the existence of a patient health problem. There may be a reason for the lack of a bowel movement for 2 days, or it might be this person's normal pattern.

A nurse develops a detailed care plan for a 16-year-old patient who is a new single mother of a premature infant. The plan includes collaborative care measures and home health care visits. When presented with the plan, the patient states, "We will be fine on our own. I don't need any more care." What would be the nurse's best response?

a) "You know your personal situation better than I do, so I will respect your wishes." b) "If you don't accept these services, your baby's health will suffer." c) "Let's take a look at the plan again and see if we can adjust it to fit your needs." d) "I'm going to assign your case to a social worker who can explain the services better." Ans: c. When a patient does not follow the care plan despite your best efforts, it is time to reassess strategy. The first objective is to identify why the patient is not following the therapy. If the nurse determines, however, that the care plan is adequate, the nurse must identify and remedy the factors contributing to the patient's noncompliance.

A student nurse tells the instructor that a patient is fine and has "no complaints." What would be the instructor's best response?

a) "You made an inference that she is fine because she has no complaints. How did you validate this?" b) "She probably just doesn't trust you enough to share what she is feeling. I'd work on developing a trusting relationship." c) "Sometimes everyone gets lucky. Why don't you try to help another patient?" d) "Maybe you should reassess the patient. She has to have a problem—why else would she be here?" Ans: a. The instructor is most likely to challenge the inference that the patient is "fine" simply because she is telling you that she has no problems. It is appropriate for the instructor to ask how the student nurse validated this inference. Jumping to the conclusion that the patient does not trust the student nurse is premature and is an invalidated inference. Answer c is wrong because it accepts the invalidated inference. Answer d is wrong because it is possible that the condition is resolving.

A friend of a nurse calls and tells the nurse that his girlfriend's father was just admitted to the hospital as a patient, and he wants the nurse to provide information about the man's condition. The friend states, "Sue seems unusually worried about her dad, but she won't talk to me and I want to be able to help her." What is the best initial response the nurse should make?

a) "You shouldn't be asking me to do this. I could be fined or even lose my job for disclosing this information." b) "Sorry, but I'm not able to give information about patients to the public—even when my best friend or a family member asks." c) "Because of HIPAA, you shouldn't be asking for this information unless the patient has authorized you to receive it! This could get you in trouble!" d) "Why do you think Sue isn't talking about her worries?" Ans: b. The nurse should immediately clarify what he or she can and cannot do. Since the primary reason for refusing to help is linked to the responsibility to protect patient privacy and confidentiality, the nurse should not begin by mentioning the real penalties linked to abuses of privacy. Finally, it is appropriate to ask about Sue and her worries, but this should be done after the nurse clarifies what he or she is able to do.

A nurse uses the classic elements of evaluation when caring for patients: (1) Interpreting and summarizing findings (2) Collecting data to determine whether evaluative criteria and standards are met (3) Documenting your judgment (4) Terminating, continuing, or modifying the plan (5) Identifying evaluative criteria and standards (what you are looking for when you evaluate—i.e., expected patient outcomes) Which item below places them in their correct sequence?

a) 1, 2, 3, 4, 5 b) 3, 2, 1, 4, 5 c) 5, 2, 1, 3, 4 d) 2, 3, 1, 4, 5 Ans: c. The five classic elements of evaluation in order are (1) identifying evaluative criteria and standards (what you are looking for when you evaluate—i.e., expected patient outcomes); (2) collecting data to determine whether these criteria and standards are met; (3) interpreting and summarizing findings; (4) documenting your judgment; and (5) terminating, continuing, or modifying the plan.

A nurse is prioritizing the following patient diagnoses according to Maslow's hierarchy of human needs:(1) Disturbed Body Image(2) Ineffective Airway Clearance(3) Spiritual Distress(4) Impaired Social InteractionWhich answer choice below lists the problems in order of highest priority to lowest priority based on Maslow's model?

a) 2, 4, 1, 3 b) 3, 1, 4, 2 c) 2, 4, 3, 1 d) 3, 2, 4, 1 Ans: a. 2, 4, 1, 3. Because basic needs must be met before a person can focus on higher ones, patient needs may be prioritized according to Maslow's hierarchy: (1) physiologic needs, (2) safety needs, (3) love and belonging needs, (4) self-esteem needs, and (5) self-actualization needs. #2 is an example of a physiologic need, #4 is an example of a love and belonging need, #1 is an example of a self-esteem need, and #3 is an example of a self-actualization need.

A nurse is documenting patient data in the medical record of a patient admitted to the hospital with appendicitis. The health care provider has ordered 10-mg morphine IV every 3 to 4 hours. Which examples of documentation of care for this patient follow recommended guidelines? Select all that apply.

a) 6/12/20 0945 Morphine 10 mg administered IV. Patient's response to pain appears to be exaggerated. M. Patrick, RN b) 6/12/20 0945 Morphine 10 mg administered IV. Patient seems to be comfortable. M. Patrick, RN c) 6/12/20 0945 30 minutes following administration of morphine 10 mg IV, patient reports pain as 2 on a scale of 1 to 10. M. Patrick, RN d) 6/12/20 0945 Patient reports severe pain in right lower quadrant. M. Patrick, RN e) 6/12/20 0945 Morphine IV 10 mg will be administered to patient every 3 to 4 hours. M. Patrick, RN f) 6/12/20 0945 Patient states she does not want pain medication despite return of pain. After discussing situation, patient agrees to medication administration. M. Patrick, RN ANS: c, d, f. The nurse should enter information in a complete, accurate, concise, current, and factual manner and indicate in each entry the date and both the time the entry was written and the time of pertinent observations and interventions. When charting, the nurse should avoid the use of stereotypes or derogatory terms as well as generalizations such as "patient's response to pain appears to be exaggerated" or "seems to be comfortable." The nurse should never document an intervention before carrying it out.

A nurse caring for culturally diverse patients in a health care provider's office is aware that patients of certain cultures are more prone to specific disease states than the general population. Which patients would the nurse screen for diabetes mellitus based on the patient's race? Select all that apply.

a) A Native American patient b) An African-American patient c) An Alaska Native d) An Asian patient e) A White patient f) A Hispanic patient Ans: a, c, e, f. Native Americans, Alaska Natives, Hispanics, and Whites are more prone to developing diabetes mellitus. African Americans are prone to hypertension, stroke, sickle cell anemia, lactose intolerance, and keloids. Asians are prone to hypertension, liver cancer, thalassemia, and lactose intolerance.

In order to provide culturally competent care, nurses must be alert to factors inhibiting sensitivity to diversity in the health care system. Which nursing actions are examples of cultural imposition? Select all that apply.

a) A hospital nurse tells a nurse's aide that patients should not be given a choice whether or not to shower or bathe daily. b) A nurse treats all patients the same whether or not they come from a different culture. c) A nurse tells another nurse that Jewish diet restrictions are just a way for them to get a special tray of their favorite foods. d) A Catholic nurse insists that a patient diagnosed with terminal bladder cancer see the chaplain in residence. e) A nurse directs interview questions to an older adult's daughter even though the patient is capable of answering them. f) A nurse refuses to care for a married gay man who is HIV positive because she is against same-sex marriage. Ans: a, d. Cultural imposition occurs when a hospital nurse tells a nurse's aide that patients should not be given a choice whether or not to shower or bathe daily, and when a Catholic nurse insists that a patient diagnosed with terminal bladder cancer see the chaplain in residence. Cultural blindness occurs when a nurse treats all patients the same whether or not they come from a different culture. Culture conflict occurs when a nurse ridicules a patient by telling another nurse that Jewish diet restrictions are just a way for Jewish patients to get a special tray of their favorite foods. When a nurse refuses to respect an older adult's ability to speak for himself or herself, or if the nurse refuses to treat a patient based on that patient's sexual orientation, the nurse is engaging in stereotyping.

A nurse performs nurse-initiated nursing actions when caring for patients in a skilled nursing facility. Which are examples of these types of interventions? Select all that apply.

a) A nurse administers 500 mg of ciprofloxacin to a patient with pneumonia. b) A nurse consults with a psychiatrist for a patient who abuses pain killers. c) A nurse checks the skin of bedridden patients for skin breakdown. d) A nurse orders a kosher meal for an orthodox Jewish patient. e) A nurse records the I&O of a patient as prescribed by his health care provider. f) A nurse prepares a patient for minor surgery according to facility protocol. Ans: c, d, f. Nurse-initiated interventions, or independent nursing actions, involve carrying out nurse-prescribed interventions resulting from their assessment of patient needs written on the nursing care plan, as well as any other actions that nurses initiate without the direction or supervision of another health care professional. Protocols and standard orders empower the nurse to initiate actions that ordinarily require the order or supervision of a health care provider. Consulting with a psychiatrist is a collaborative intervention.

A nurse on a busy surgical unit relies on informal planning to provide appropriate nursing responses to patients in a timely manner. What are examples of this type of planning? Select all that apply.

a) A nurse sits down with a patient and prioritizes existing diagnoses. b) A nurse assesses a woman for postpartum depression during routine care. c) A nurse plans interventions for a patient who is diagnosed with epilepsy. d) A busy nurse takes time to speak to a patient who received bad news. e) A nurse reassesses a patient whose PRN pain medication is not working. f) A nurse coordinates the home care of a patient being discharged. Ans: b, d, e. Informal planning is a link between identifying a patient's strength or problem and providing an appropriate nursing response. This occurs, for example, when a busy nurse first recognizes postpartum depression in a patient, takes time to assess a patient who received bad news about tests, or reassesses a patient for pain. Formal planning involves prioritizing diagnoses, formally planning interventions, and coordinating the home care of a patient being discharged.

A nurse is using the SOAP format to document care of a patient who is diagnosed with type 2 diabetes. Which source of information would be the nurse's focus when completing this documentation?

a) A patient problem list b) Narrative notes describing the patient's condition c) Overall trends in patient status d) Planned interventions and patient outcomes Ans: a. The SOAP format (Subjective data, Objective data, Assessment, Plan) is used to organize entries in the progress notes of a POMR. When using the SOAP format, the problem list at the front of the chart alerts all caregivers to patient priorities. Narrative notes allow nurses to describe a condition, situation, or response in their own terms. Overall trends in patient status can be seen immediately when using CBE, not SOAP charting. Planned interventions and patient-expected outcomes are the focus of the case management model.

The nurse collects objective and subjective data when conducting patient assessments. Which patient situations are examples of subjective data? Select all that apply.

a) A patient tells the nurse that she is feeling nauseous.A patient's ankles are swollen. b) A patient tells the nurse that she is nervous about her test results. c) A patient complains that the skin on her arms is tingling. d) A patient rates his pain as a 7 on a scale of 1 to 10.A patient vomits after eating supper. Ans: a, c, d, e. Subjective data are information perceived only by the affected person; these data cannot be perceived or verified by another person. Examples of subjective data are feeling nervous, nauseated, tingling, and experiencing pain. Objective data are observable and measurable data that can be seen, heard, or felt by someone other than the person experiencing them. Examples of objective data are an elevated temperature reading (e.g., 101°F), edema, and vomiting.

A nurse is collecting more patient data to confirm a patient diagnosis of emphysema. This is an example of formulating what type of diagnosis?

a) Actual b) Possible c) Risk d) Collaborative Ans: b. An intervention for a possible diagnosis is to collect more patient data to confirm or rule out the problem. An intervention for an actual diagnosis is to reduce or eliminate contributing factors to the diagnosis. Interventions for a risk diagnosis focus on reducing or eliminating risk factors, and interventions for collaborative problems focus on monitoring for changes in status and managing these changes with nurse- and physician-prescribed interventions.

A nurse makes a clinical judgment that an African American man in a stressful job is more vulnerable to developing hypertension than a White man in the same or a similar situation. The nurse has formulated what type of nursing diagnosis?

a) Actual b) Risk c) Possible d) Wellness Ans: b. A clinical judgment that an individual, family, or community is more vulnerable to develop the problem than others in the same or similar situation is a Risk nursing diagnosis.

A nurse is about to perform pin site care for a patient who has a halo traction device installed. What is the FIRST nursing action that should be taken prior to performing this care?

a) Administer pain medication. b) Reassess the patient. c) Prepare the equipment. d) Explain the procedure to the patient. Ans: b. Before implementing any nursing action, the nurse should reassess the patient to determine whether the action is still needed. Then the nurse may collect the equipment, explain the procedure, and, if necessary, administer pain medications.

A nurse is looking for trends in a postoperative patient's vital signs. Which documents would the nurse consult first?

a) Admission sheet b) Admission nursing assessment c) Flow sheet d) Graphic record Ans: d. While one recording of vital signs should appear on the admission nursing assessment, the best place to find sequential recordings that show a pattern or trend is the graphic record. The admission sheet does not include vital sign documentation, and neither does the flow sheet.

A nurse is preparing a clinical outcome for a patient who is an avid runner and who is recovering from a stroke that caused right-sided paresis. What is an example of this type of outcome?

a) After receiving 3 weeks of physical therapy, patient will demonstrate improved movement on the right side of her body. b) By 8/15/20, patient will be able to use right arm to dress, comb hair, and feed herself. c) Following physical therapy, patient will begin to gradually participate in walking/running events. d) By 8/15/20, patient will verbalize feeling sufficiently prepared to participate in running events. Ans: a. Clinical outcomes describe the expected status of health issues at certain points in time, after treatment is complete. Functional outcomes (b) describe the person's ability to function in relation to the desired usual activities. Quality-of-life outcomes (c) focus on key factors that affect someone's ability to enjoy life and achieve personal goals. Affective outcomes (d) describe changes in patient values, beliefs, and attitudes.

A new RN is being oriented to a nursing unit that is currently understaffed and is told that the UAPs have been trained to obtain the initial nursing assessment. What is the best response of the new RN?

a) Allow the UAPs to do the admission assessment and report the findings to the RN. b) Do his or her own admission assessments but don't interfere with the practice if other professional RNs seem comfortable with the practice. c) Tell the charge nurse that he or she chooses not to delegate the admission assessment until further clarification is received from administration. d) Contact his or her labor representative to report this practice to the state board of nursing. Ans: c. The nurse should not delegate this nursing admission assessment because only nurses can perform this intervention. The nurse should seek clarification for this policy from the nursing administration.

A nurse observes that a patient who has cataracts is sitting closer to the television than usual. Which alteration would the nurse suspect is causing this patient behavior?

a) Altered stimulation b) Altered sensory reception c) Altered nerve impulse conduction d) Altered impulse translation Ans: b. Cataracts are interfering with the patient's ability to receive visual stimuli, causing altered sensory reception. The nature of incoming stimuli (e.g., environmental stimuli), the conduction of nerve impulses, and the translation of incoming impulses in the brain are not problematic in this situation.

Which patient would a nurse assess as being at greatest risk for sensory deprivation?

a) An older adult confined to bed at home after a stroke b) An adolescent in an oncology unit working on homework supplied by friends c) A woman in labor d) A toddler in a playroom awaiting same-day surgery Ans: a. The patient confined to bed rest at home is at risk for greatly reduced environmental stimuli. All of the other patients are in environments in which environmental stimuli are at least adequate.

An older patient has a severe visual deficit related to glaucoma. Which nursing action would be appropriate when providing care for this patient?

a) Assist the patient to ambulate by walking slightly behind her and grasping the arm. b) Concentrate on the patient's sense of sight and limit diversions that involve other senses. c) Stay outside of the patient's field of vision when performing personal hygiene for her. d) Indicate to the patient when the conversation has ended and when the nurse is leaving the room Ans: d. When caring for a patient who has a visual deficit, the nurse should indicate when the conversation is over and when he or she is leaving the room. When assisting with ambulation, the nurse should walk slightly ahead of (rather than behind) the patient and allow her to grasp the nurse's arm. The nurse should provide, rather than limit, diversions using other senses, and stay in the person's field of vision if she has partial or reduced peripheral vision.

A nurse is attempting to improve care on the pediatric ward of a hospital. Which nursing improvements might the nurse employ when following the recommendations of the Institute of Medicine's Committee on Quality of Health Care in America? Select all that apply.

a) Basing patient care on continuous healing relationships b) Customizing care to reflect the competencies of the staff c) Using evidence-based decision makingHaving a charge nurse as the source of control d) Using safety as a system priority e) Recognizing the need for secrecy to protect patient privacy Ans: a, c, e. Care should be based on continuous healing relationships and evidence-based decision making. Customization should be based on patient needs and values with the patient as the source of control. Safety should be used as a system priority, and the need for transparency should be recognized.

A nurse is caring for a patient who presents with labored respirations, productive cough, and fever. What would be appropriate nursing diagnoses for this patient? Select all that apply.

a) Bronchial pneumonia b) Impaired gas exchange c) Ineffective airway clearance d) Potential complication: sepsis e) Infection related to pneumonia f) Risk for septic shock Ans: b, c, f. Nursing diagnoses are actual or potential health problems that can be prevented or resolved by independent nursing interventions, such as impaired gas exchange, ineffective airway clearance, or risk for septic shock. Bronchial pneumonia and infection are medical diagnoses, and "potential complication: sepsis" is a collaborative problem.

Nurses use the NIC Taxonomy structure as a resource when planning nursing care for patients. What information is found in this structure?

a) Case studies illustrating a complete set of activities that a nurse performs to carry out nursing interventions b) Nursing interventions, each with a label, a definition, and a set of activities that a nurse performs to carry it out, with a short list of background readings c) A complete list of nursing diagnoses, outcomes, and related nursing activities for each nursing intervention d) A complete list of reimbursable charges for each nursing intervention Ans: b. The NIC Taxonomy lists nursing interventions, each with a label, a definition, a set of activities that a nurse performs to carry it out, and a short list of background readings. It does not contain case studies, diagnoses, or charges.

A new nurse manager at a small hospital is interested in achieving Magnet status. Which action would help the hospital to achieve this goal?

a) Centralizing the decision-making process b) Promoting self-governance at the unit level c) Deterring professional autonomy to promote teamwork d) Promoting evidence-based practice over innovative nursing practice Ans: b. Magnet hospitals use a decentralized decision-making process, self-governance at the unit level, and respect for and acknowledgment of professional autonomy. In Magnet hospitals, 14 characteristics, the Forces of Magnetism, have been recognized that identify quality patient care, excellent nursing care, and innovations in professional nursing practice.

A nurse is assessing a patient who is diagnosed with anorexia. Following the assessment, the nurse recommends that the patient meet with a nutritionist. This action best exemplifies the use of:

a) Clinical judgment b) Clinical reasoning c) Critical thinking d) Blended competencies Ans: a. Although all the options refer to the skills used by nurses in practice, the best choice is clinical judgment as it refers to the result or outcome of critical thinking or clinical reasoning—in this case, the recommendation to meet with a nutritionist. Clinical reasoning usually refers to ways of thinking about patient care issues (determining, preventing, and managing patient problems). Critical thinking is a broad term that includes reasoning both outside and inside of the clinical setting. Blended competencies are the cognitive, technical, interpersonal, and ethical and legal skills combined with the willingness to use them creatively and critically when working with patients.

A nurse writes the following outcome for a patient who is trying to lose weight: "The patient can explain the relationship between weight loss, increased exercise, and decreased calorie intake." This is an example of what type of outcome?

a) Cognitive b) Psychomotor c) Affective d) Physical changes Ans: a. Cognitive outcomes involve increases in patient knowledge; psychomotor outcomes describe the patient's achievement of new skills; affective outcomes pertain to changes in patient values, beliefs, and attitudes; and physical changes are actual bodily changes in the patient (e.g., weight loss, increased muscle tone).

A nurse writes the following outcome for a patient who is trying to stop smoking: "The patient values a healthy body sufficiently to stop smoking." This is an example of what type of outcome?

a) Cognitive b) Psychomotor c) Affective d) Physical changes Ans: c. Affective outcomes pertain to changes in patient values, beliefs, and attitudes. Cognitive outcomes involve increases in patient knowledge; psychomotor outcomes describe the patient's achievement of new skills; physical changes are actual bodily changes in the patient (e.g., weight loss, increased muscle tone).

A nurse manager of a busy cardiac unit observes disagreements between the RNs and the LPNs related to schedules and nursing responsibilities. At a staff meeting, the manager compliments all the nurses on a job well done and points out that expected goals and outcomes for the month have been met. The nurse concludes the meeting without addressing the disagreements between the two groups of nurses. Which conflict resolution strategy is being employed by this manager?

a) Collaborating b) Competing c) Compromising d) Smoothing Ans: d. The manager who resolves conflict by complimenting the parties involved and focusing on agreement rather than disagreement is using smoothing to reduce the emotion in the conflict. The original conflict is rarely resolved with this technique. Collaborating is a joint effort to resolve the conflict with a win-win solution. All parties set aside previously determined goals, determine a priority common goal, and accept mutual responsibility for achieving this goal. Competing results in a win for one party at the expense of the other group. Compromising occurs when both parties relinquish something of equal value.

A nurse is counseling a patient who refuses to look at or care for a new colostomy. The patient tells the nurse, "I don't care what I look like anymore, I don't even feel like washing my hair, let alone changing this bag." The nurse diagnoses Altered Health Maintenance. This is an example of what type of problem?

a) Collaborative problem b) Interdisciplinary problem c) Medical problem d) Nursing problem Ans: d. Altered Health Maintenance is a nursing problem, because the diagnosis describes a problem that can be treated by nurses within the scope of independent nursing practice. Collaborative and interdisciplinary problems require a teamwork approach with other health care professionals to resolve the problem. A medical problem is a traumatic or disease condition validated by medical diagnostic studies.

The nurse records a patient's blood pressure as 148/100. What is the priority action of the nurse when determining the significance of this reading?

a) Compare this reading to standards. b) Check the taxonomy of nursing diagnoses for a pertinent label. c) Check a medical text for the signs and symptoms of high blood pressure. d) Consult with colleagues. Ans: a. A standard, or a norm, is a generally accepted rule, measure, pattern, or model to which data can be compared in the same class or category. For example, when determining the significance of a patient's blood pressure reading, appropriate standards include normative values for the patient's age group, race, and illness category. Deviation from an appropriate norm may be the basis for writing a diagnosis.

The nurse practitioner is performing a short assessment of a newborn who is displaying signs of jaundice. The nurse observes the infant's skin color and orders a test for bilirubin levels to report to the primary care provider. What type of assessment has this nurse performed?

a) Comprehensive b) Initial c) Time-lapsed d) Quick priority Ans: d. Quick priority assessments (QPAs) are short, focused, prioritized assessments nurses do to gain the most important information they need to have first. The comprehensive initial assessment is performed shortly after the patient is admitted to a health care facility or service. The time-lapsed assessment is scheduled to compare a patient's current status to baseline data obtained earlier.

A student health nurse is counseling a college student who wants to lose 20 lb. The nurse develops a plan to increase the student's activity level and decrease her consumption of the wrong types of foods and excess calories. The nurse plans to evaluate the student's weight loss monthly. When the student arrives for her first "weigh-in," the nurse discovers that instead of the projected weight loss of 5 lb, the student has lost only 1 lb. Which is the BEST nursing response?

a) Congratulate the student and continue the care plan. b) Terminate the care plan since it is not working. c) Try giving the student more time to reach the targeted outcome. d) Modify the care plan after discussing possible reasons for the student's partial success. Ans: d. Since the student has only partially met her outcome, the nurse should first explore the factors making it difficult for her to reach her outcome and then modify the care plan. It would not be appropriate to continue the plan as it is since it is not working, and it is premature to terminate the care plan since the student has not met her targeted outcome. The student may need more than just additional time to reach her outcome.

A nurse manager who is attempting to institute the SBAR process to communicate with health care providers and transfer patient information to other nurses is meeting staff resistance to the change. Which action would be most effective in approaching this resistance?

a) Containing the anxiety in a small group and moving forward with the initiative b) Explaining the change and listing the advantages to the person and the organization c) Reprimanding those who oppose the new initiative and praising those who willingly accept the change d) Introducing the change quickly and involving the staff in the implementation of the change Ans: b. Change is ubiquitous, as is resistance to change. The manager should explain the proposed change to all affected, list the advantages of the proposed change for all parties, introduce the change gradually, and involve everyone affected by the change in the design and implementation of the process. The manager should not use the reward/punishment style to overcome resistance to change.

A nurse notes that a shift report states that a patient has no special skin care needs. The nurse is surprised to observe reddened areas over bony prominences during the patient bath. What nursing action is appropriate?

a) Correct the initial assessment form. b) Redo the initial assessment and document current findings. c) Conduct and document an emergency assessment. d) Perform and document a focused assessment of skin integrity. Ans: d. Perform and document a focused assessment on skin integrity since this is a newly identified problem. The initial assessment stands as is and cannot be redone or corrected. This is not a life-threatening event; therefore, there is no need for an emergency assessment.

The nurse practitioner sees patients in a community clinic that is located in a predominately White neighborhood. After performing assessments on the majority of the patients visiting the clinic, the nurse notes that many of the minority groups living within the neighborhood have lost the cultural characteristics that made them different. What is the term for this process?

a) Cultural assimilation b) Cultural imposition c) Culture shock d) Ethnocentrism Ans: a. When minority groups live within a dominant group, many members lose the cultural characteristics that once made them different in a process called assimilation. Cultural imposition occurs when one person believes that everyone should conform to his or her own belief system. Culture shock occurs when a person is placed in a different culture perceived as strange, and ethnocentrism is the belief that the ideas, beliefs, and practices of one's own cultural group are best, superior, or most preferred to those of other groups.

A nurse is telling a new mother from Africa that she shouldn't carry her baby in a sling created from a large rectangular cloth. The African woman tells the nurse that everyone in Mozambique carries babies this way. The nurse believes that bassinets are safer for infants. This nurse is displaying what cultural bias?

a) Cultural imposition b) Clustering c) Cultural competency d) Stereotyping Ans: a. The nurse is trying to impose her belief that bassinets are preferable to baby slings on the African mother—in spite of the fact that African women have safely carried babies in these slings for years.

A nurse states, "That patient is 78 years old—too old to learn how to change a dressing." What is the nurse demonstrating?

a) Cultural imposition b) Clustering c) Cultural competency d) Stereotyping Ans: d. Stereotyping is assuming that all members of a group are alike. This is not an example of cultural competence nor is the nurse imposing her culture on the patient. Clustering is not an applicable concept.

A nurse is using information from informatics technology that is synthesized so that relationships between lung cancer diagnoses and smoking are identified. What part of "DIKW" does this represent?

a) Data b) Information c) Knowledge d) Wisdom Ans: c. Knowledge is Information that is synthesized so that relationships are identified. Data refer to discrete entities that are described without interpretation. Information is data that have been interpreted, organized, or structured. Wisdom is the appropriate use of knowledge to manage and solve human problems.

A nurse is using informatics technology to decide which patients may be at risk for readmission. What is the term for this type of analytic?

a) Data visualization b) Predictive analytics c) Big data d) Data recall Ans: b. Predictive analytics encompasses a variety of statistical techniques that analyze current and historical facts to make predictions about future or otherwise unknown events. In health care, this is used by organizations to attempt to identify patients who are at risk for readmission so case managers can intervene. Data visualization is the presentation of data in a pictorial or graphical format for analysis. Big data comprises the accumulation of health care-related data from various sources, combined with new technologies that allow for the transformation of data to information, to knowledge, and ultimately to wisdom. Data recall is not a technical term for analytics.

Nurses incorporate telecare in patient care plans. Which services are MOST representative of this technologic advance? Select all that apply.

a) Diagnostic testing b) Easy access to specialists d) Health and fitness apps e) Early warning and detection technologies f) Digital medication reminder systems g) Monitoring of progress following treatment Ans: b, c, d. Telecare generally refers to technology that allows consumers to stay safe and independent in their own homes. It may include consumer-oriented health and fitness apps, sensors and tools that connect consumers with family members or other caregivers, exercise tracking tools, digital medication reminder systems, and early warning and detection technologies. Telemedicine involves the use of telecommunications technologies to support the delivery of all types of medical, diagnostic, and treatment-related services, usually by physicians or nurse practitioners. Examples include conducting diagnostic tests, monitoring a patient's progress after treatment or therapy, and facilitating access to specialists that are not located in the same place as the patient.

A nurse is documenting the care given to a patient diagnosed with an osteosarcoma, whose right leg was amputated. The nurse accidentally documents that a dressing changed was performed on the left leg. What would be the best action of the nurse to correct this documentation?

a) Erase or use correcting fluid to completely delete the error. b) Mark the entry "mistaken entry"; add correct information; date and initial. c) Use a permanent marker to block out the mistaken entry and rewrite it. e) Remove the page with the error and rewrite the data on that page correctly. Ans: b. The nurse should not use dittos, erasures, or correcting fluids when correcting documentation; block out a mistake with a permanent marker; or remove a page with an error and rewrite the data on a new page. To correct an error after it has been entered, the nurse should mark the entry "mistaken entry," add the correct information, and date and initial the entry. If the nurse records information in the wrong chart, the nurse should write "mistaken entry—wrong chart" and sign off. The nurse should follow similar guidelines in electronic records.

A patient has an order for an analgesic medication to be given PRN. When would the nurse administer this medication?

a) Every 3 hours b) Every 4 hours c) Daily d) As needed Ans: d. PRN means "as needed"—not every 3 hours, every 4 hours, or once daily.

The nurse practices using critical thinking indicators (CTIs) when caring for patients in the hospital setting. The best description of CTIs is:

a) Evidence-based descriptions of behaviors that demonstrate the knowledge that promotes critical thinking in clinical practice b) Evidence-based descriptions of behaviors that demonstrate the knowledge and skills that promote critical thinking in clinical practice c) Evidence-based descriptions of behaviors that demonstrate the knowledge, characteristics, and skills that promote critical thinking in clinical practice d) Evidence-based descriptions of behaviors that demonstrate the knowledge, characteristics, standards, and skills that promote critical thinking in clinical practice Ans: c. Evidence-based descriptions of behaviors that demonstrate the knowledge, characteristics, and skills that promote critical thinking in clinical practice.

A student nurse is organizing clinical responsibilities for a patient who is diabetic and is being treated for foot ulcers. The patient tells the student, "I need to have my hair washed before I can do anything else today; I'm ashamed of the way I look." The patient's needs include diagnostic testing, dressing changes, meal planning and counseling, and assistance with hygiene. How would the nurse best prioritize this patient's care?

a) Explain to the patient that there is not enough time to wash her hair today because of her busy schedule. b) Schedule the testing and meal planning first and complete hygiene as time permits. c) Perform the dressing changes first, schedule the testing and counseling, and complete hygiene last. d) Arrange to wash the patient's hair first, perform hygiene, and schedule diagnostic testing and counseling. Ans: d. As long as time constraints permit, the most important priorities when scheduling nursing care are priorities identified by the patient as being most important. In this case, washing the patient's hair and assisting with hygiene puts the patient first and sets the tone for an effective nurse-patient partnership.

A nurse assessing an 8-month-old infant suspects the infant is experiencing sensory deprivation related to inadequate parenting. Since this assessment, both parents have attended parenting classes. However, both parents work while the infant stays with a grandparent, who has reduced vision. The parents provide appropriate stimulation in the evening. At an evaluation conference at the age of 11 months, the infant lies on the floor, rocking back and forth and has a dull facial expression with few vocalizations. Which nursing action would be appropriate for this patient and family?

a) Explore why the infant's parents lack motivation to provide necessary stimulation. b) Remove the infant from the grandmother's care as the child has not progressed. c) Suggest counseling since the infant's sensory deprivation is still severe. d) No action is needed, as this is normal behavior for an 11-month-old infant. Ans: c. Although the data show that the parents have been motivated to improve their parenting skills, it is clear from the data that the infant's sensory deprivation is still severe. The data suggest that the grandmother is not improving the infant's care, but there is nothing to suggest that she is unable to do so if shown how.

A nurse is using the ESFT model to understand a patient's conception of a diagnosis of chronic obstructive pulmonary disease (COPD). Which interview question would be MOST appropriate to assess the E aspect of this model—Explanatory model of health and illness?

a) How do you get your medications? b) How does having COPD affect your lifestyle? c) Are you concerned about the side effects of your medications? d) Can you describe how you will take your medications? Ans: b. The ESFT model guides providers in understanding a patient's explanatory model (a patient's conception of her or his illness), social and environmental factors, and fears and concerns, and also guides providers in contracting for therapeutic approaches. Asking the questions: "How does having COPD affect your lifestyle?" explores the explanatory model, "How do you get your medications?" refers to the social and environmental factor, "Are you concerned about the side effects of your medications?" addresses fears and concerns, and "Can you describe how you will take your medications?" involves therapeutic contracting.

An older adult in a long-term care facility walked out the door unobserved and was lost for several hours. Upon assessment, the nurse notes that the patient is confused and documents: chronic sensory deprivation related to the effects of aging. Which interventions would be most effective for this patient? Select all that apply.

a) Ignore the patient's confusion, or go along with it to prevent embarrassment. b) Reduce the number and type of stimuli in the patient's room. c) Orient the patient to time, place, and person frequently. d) Provide daily contact with children, community people, and pets. e) Decrease background or loud noises in the environment. f) Provide a radio and television in the patient's room. Ans: c, d, f. Even if well motivated, ignoring a patient's confusion to prevent embarrassment may be dangerous, as it was in this case in which the appropriate safety precautions were never implemented. Reducing the type of stimuli in the room and decreasing environmental noise is appropriate for a patient who is experiencing sensory overload. The other options are related to sensory deprivation and are appropriate for this patient.

The nurse is surprised to detect an elevated temperature (102°F) in a patient scheduled for surgery. The patient has been afebrile and shows no other signs of being febrile. What is the priority nursing action?

a) Inform the charge nurse. b) Inform the surgeon. c) Validate the finding. d) Document the finding. Ans: c. The nurse should first validate the finding if it is unusual, deviates from normal, and is unsupported by other data. Should the initial recording prove to be in error, it would have been premature to notify the charge nurse or the surgeon. The nurse should be sure that all data recorded are accurate; thus, all data should be validated before documentation if there are any doubts about accuracy.

The nurse is helping a patient turn in bed and notices the patient's heels are red. The nurse places the patient on precautions for skin breakdown. This is an example of what type of planning?

a) Initial planning b) Standardized planning c) Ongoing planning d) Discharge planning Ans: c. Ongoing planning is problem oriented and has as its purpose keeping the plan up to date as new actual or potential problems are identified. Initial planning addresses each problem listed in the prioritized nursing diagnoses and identifies appropriate patient goals and the related nursing care. Standardized care plans are prepared care plans that identify the nursing diagnoses, outcomes, and related nursing interventions common to a specific population or health problem. During discharge planning, the nurse uses teaching and counseling skills effectively to help the patient and family develop sufficient knowledge of the health problem and the therapeutic regimen to carry out necessary self-care behaviors competently at home.

An experienced nurse tells a beginning nurse not to bother studying too hard, since most clinical reasoning becomes "second nature" and "intuitive" once you start practicing. What thinking below should underlie the beginning nurse's response?

a) Intuitive problem solving comes with years of practice and observation, and novice nurses should base their care on scientific problem solving. b) For nursing to remain a science, nurses must continue to be vigilant about stamping out intuitive reasoning. c) The emphasis on logical, scientific, evidence-based reasoning has held nursing back for years; it is time to champion intuitive, creative thinking! d) It is simply a matter of preference; some nurses are logical, scientific thinkers, and some are intuitive, creative thinkers. Ans: a. Beginning nurses must use nursing knowledge and scientific problem solving as the basis of care they give; intuitive problem solving comes with years of practice and observation. If the beginning nurse has an intuition about a patient, that information should be discussed with the faculty member, preceptor, or supervisor. Answer b is incorrect because there is a place for intuitive reasoning in nursing, but it will never replace logical, scientific reasoning. Critical thinking is contextual and changes depending on the circumstances, not on personal preference.

A nurse uses critical thinking skills to focus on the care plan of an older adult who has dementia and needs placement in a long-term care facility. Which statements describe characteristics of this type of critical thinking applied to clinical reasoning? Select all that apply.

a) It functions independently of nursing standards, ethics, and state practice acts. b) It is based on the principles of the nursing process, problem solving, and the scientific method. c) It is driven by patient, family, and community needs as well as nurses' needs to give competent, efficient care. d) It is not designed to compensate for problems created by human nature, such as medication errors. e) It is constantly re-evaluating, self-correcting, and striving for improvement. f) It focuses on the big picture rather than identifying the key problems, issues, and risks involved with patient care. Ans: b, c, e. Critical thinking applied to clinical reasoning and judgment in nursing practice is guided by standards, policies and procedures, and ethics codes. It is based on principles of nursing process, problem solving, and the scientific method. It carefully identifies the key problems, issues, and risks involved, and is driven by patient, family, and community needs, as well as nurses' needs to give competent, efficient care. It also calls for strategies that make the most of human potential and compensate for problems created by human nature. It is constantly re-evaluating, self-correcting, and striving to improve (Alfaro-LeFevre, 2014).

A nurse is teaching a novice nurse how to provide care for patients in a culturally diverse community health clinic. Although all these actions are recommended, which one is MOST basic to providing culturally competent care?

a) Learning the predominant language of the community b) Obtaining significant information about the community c) Treating each patient at the clinic as an individual d) Recognizing the importance of the patient's family Ans: c. In all aspects of nursing, it is important to treat each patient as an individual. This is also true in providing culturally competent care. This basic objective can be accomplished by learning the predominant language in the community, researching the patient's culture, and recognizing the influence of family on the patient's life.

In a group home in which most patients have slight to moderate visual or hearing impairment and some are periodically confused, what would be a nurse's first priority in caring for sensory concerns?

a) Maintaining safety and preventing sensory deterioration b) Insisting that every patient participate in as many self-care activities as possible c) Emphasizing and reinforcing individual patient strengths d) Encouraging reminiscence and life review in groups Ans: a. Safety is a basic physiologic need that must be met before higher-level needs—such as love and belonging, self-esteem, and self-actualization—can be met.

A nurse is performing an initial comprehensive assessment of a patient admitted to a long-term care facility from home. The nurse begins the assessment by asking the patient, "How would you describe your health status and well-being?" The nurse also asks the patient, "What do you do to keep yourself healthy?" Which model for organizing data is this nurse following?

a) Maslow's human needs b) Gordon's functional health patterns c) Human response patterns d) Body system model Ans: b. Gordon's functional health patterns begin with the patient's perception of health and well-being and progress to data about nutritional-metabolic patterns, elimination patterns, activity, sleep/rest, self-perception, role relationship, sexuality, coping, and values/beliefs. Maslow's model is based on the human needs hierarchy. Human responses include exchanging, communicating, relating, valuing, choosing, moving, perceiving, knowing, and feeling. The body system model is based on the functioning of the major body systems.

A new nurse who is being oriented to the subacute care unit is expected to follow existing standards when providing patient care. Which nursing actions are examples of these standards? Select all that apply.

a) Monitoring patient status every hour b) Using intuition to troubleshoot patient problems c) Turning a patient on bed rest every 2 hours d) Becoming a nurse mentor to a student nurse e) Administering pain medication ordered by the physician f) Becoming involved in community nursing events Ans: a, c, e. Standards are the levels of performance accepted and expected by the nursing staff or other health care team members. They are established by authority, custom, or consent. Standards would include monitoring patient status every hour, turning a patient on bed rest every 2 hours, and administering pain medication ordered by the physician. Using intuition to troubleshoot patient problems, becoming a nurse mentor to a student nurse, and becoming involved in community nursing events are not patient care standards.

When may a health institution release a PHI for purposes other than treatment, payment, and routine health care operations, without the patient's signed authorization? Select all that apply.

a) News media are preparing a report on the condition of a patient who is a public figure. b) Data are needed for the tracking and notification of disease outbreaks. c) Protected health information is needed by a coroner. d) Child abuse and neglect are suspected. e) Protected health information is needed to facilitate organ donation. f) The sister of a patient with Alzheimer's disease wants to help provide care. Ans: b, c, d, e. According to the HIPAA, a health institution is not required to obtain written patient authorization to release PHI for tracking disease outbreaks, infection control, statistics related to dangerous problems with drugs or medical equipment, investigation and prosecution of a crime, identification of victims of crimes or disaster, reporting incidents of child abuse, neglect or domestic violence, medical records released according to a valid subpoena, PHI needed by coroners, medical examiners, and funeral directors, PHI provided to law enforcement in the case of a death from a potential crime, or facilitating organ donations. Under no circumstance can a nurse provide information to a news reporter without the patient's express authorization. An authorization form is still needed to provide PHI for a patient who has Alzheimer's disease.

After assessing a patient who is recovering from a stroke in a rehabilitation facility, a nurse interprets and analyzes the patient data. Which of the four basic conclusions has the nurse reached when identifying the need to collect more data to confirm a diagnosis of situational low self-esteem?

a) No problem b) Possible problem c) Actual nursing diagnosis d) Clinical problem other than nursing diagnosis Ans: b. When a possible problem exists, such as situational low self-esteem related to effects of stroke, the nurse must collect more data to confirm or disprove the suspected problem. The conclusion "no problem" means no nursing response is indicated. When an actual problem exists, the nurse begins planning, implementing, and evaluating care to prevent, reduce, or resolve the problem. A clinical problem other than nursing diagnosis requires that the nurse consult with the appropriate health care professional to work collaboratively on the problem.

Which action would be most important for a nurse to include in the care plan for a patient diagnosed with presbycusis?

a) Obtaining large-print written material b) Speaking distinctly, using lower frequencies c) Decreasing tactile stimulation d) Initiating a safety program to prevent falls Ans: b. Presbycusis is a normal loss of hearing as a result of the aging process. Speaking distinctly in lower frequencies is indicated. Obtaining large-print written material is appropriate for visual alterations. Decreasing tactile stimulation is appropriate for a patient with an alteration in touch, and initiating a safety program to prevent falls is appropriate for a patient experiencing kinesthetic alterations.

A nurse is caring for a patient who is receiving fluids for dehydration. Which outcome for this patient is correctly written?

a) Offer the patient 60-mL fluid every 2 hours while awake. b) During the next 24-hour period, the patient's fluid intake will total at least 2,000 mL. c) Teach the patient the importance of drinking enough fluids to prevent dehydration by 1/15/20. d) At the next visit on 12/23/20, the patient will know that he should drink at least 3 L of water per day. Ans: b. The outcomes in (a) and (c) make the error of expressing the patient goal as a nursing intervention. Incorrect: "Offer the patient 60-mL fluid every 2 hours while awake." Correct: "The patient will drink 60-mL fluid every 2 hours while awake, beginning 1/3/20." The outcome in (d) makes the error of using verbs that are not observable and measurable. Verbs to be avoided when writing outcomes include "know," "understand," "learn," and "become aware."

A nurse is caring for a patient who has complications related to type 2 diabetes mellitus. The nurse researches new procedures to care for foot ulcers when developing a care plan for this patient. Which QSEN competency does this action represent?

a) Patient-centered care b) Evidence-based practice c) Quality improvement d) Informatics Ans: c. Quality improvement involves routinely updating nursing policies and procedures. Providing patient-centered care involves listening to the patient and demonstrating respect and compassion. Evidence-based practice is used when adhering to internal policies and standardized skills. The nurse is employing informatics by using information and technology to communicate, manage knowledge, and support decision making.

A school nurse notices that a student is losing weight and decides to perform a focused nutritional assessment to rule out an eating disorder. What is the nurse's best action?

a) Perform the focused assessment as this is an independent nurse-initiated intervention. b) Request an order from Jill's physician since this is a physician-initiated intervention. c) Request an order from Jill's physician since this is a collaborative intervention. d) Request an order from the nutritionist since this is a collaborative intervention. Ans: a. Performing a focused assessment is an independent nurse-initiated intervention; thus the nurse does not need an order from the physician or the nutritionist.

An RN working on a busy hospital unit delegates patient care to UAPs. Which patient care could the nurse most likely delegate to a UAP safely? Select all that apply.

a) Performing the initial patient assessments b) Making patient beds c) Giving patients bed baths d) Administering patient medications e) Ambulating patients f) Assisting patients with meals Ans: b, c, e, f. Performing the initial patient assessment and administering medications are the responsibility of the RN. In most cases, patient hygiene, bed-making, ambulating patients, and helping to feed patients can be delegated to a UAP.

A nurse is using a concept map care plan to devise interventions for a patient with sickle cell anemia. What is the BEST description of the "concepts" that are being diagrammed in this plan?

a) Protocols for treating the patient problem b) Standardized treatment guidelines c) The nurse's ideas about the patient problem and treatment d) Clinical pathways for the treatment of sickle cell anemia Ans: c. A concept map care plan is a diagram of patient problems and interventions. The nurse's ideas about patient problems and treatments are the "concepts" that are diagrammed. These maps are used to organize patient data, analyze relationships in the data, and enable the nurse to take a holistic view of the patient's situation. Answers (a) and (b) are incomplete because the concepts being diagrammed may include protocols and standardized treatment guidelines but the patient problems are also diagrammed concepts. Clinical pathways are tools used in case management to communicate the standardized, interdisciplinary care plan for patients.

After one nursing unit with an excellent safety record meets to review the findings of the audit, the nurse manager states, "We're doing well, but we can do better! Who's got an idea to foster increased patient well-being and satisfaction?" This is an example of leadership that values:

a) Quality assurance b) Quality improvement c) Process evaluation e) Outcome evaluation Ans: b. Unlike quality assurance, quality improvement is internally driven, focuses on patient care rather than organizational structure, focuses on processes rather than people, and has no end points. Its goal is improving quality rather than assuring quality. Process evaluation and outcome evaluation are types of quality-assurance programs.

A quality-assurance program reveals a higher incidence of falls and other safety violations on a particular unit. A nurse manager states, "We'd better find the people responsible for these errors and see if we can replace them." This is an example of:

a) Quality by inspection b) Quality by punishment c) Quality by surveillance d) Quality by opportunity Ans: a. Quality by inspection focuses on finding deficient workers and removing them. Quality as opportunity focuses on finding opportunities for improvement and fosters an environment that thrives on teamwork, with people sharing the skills and lessons they have learned. Quality by punishment and quality by surveillance are not quality-assurance methods used in the health care field.

A nurse assesses a patient and formulates the following nursing diagnosis: Risk for Impaired Skin Integrity related to prescribed bed rest as evidenced by reddened areas of skin on the heels and back. Which phrase represents the etiology of this diagnostic statement?

a) Risk for Impaired Skin Integrity b) Related to prescribed bed rest c) As evidenced by d) As evidenced by reddened areas of skin on the heels and back Ans: b. "Related to prescribed bed rest" is the etiology of the statement. The etiology identifies the contributing or causative factors of the problem. "Risk for Impaired Skin Integrity" is the problem, and "as evidenced by reddened areas of skin on the heels and back" are the defining characteristics of the problem.

A nurse is diagnosing an 11-year-old student following a physical assessment. The nurse notes that the student's grades have dropped, she has difficulty completing her work on time, and she frequently rubs her eyes and squints. Her visual acuity on a Snellen's eye chart is 160/20. Based on this assessment data, which alteration would the nurse document for this patient?

a) Self-care deficit b) Altered Role Performance (Student) c) Disturbed Body Image d) Delayed Growth and Development Ans: b. An important role for an 11 year old is that of student. Her impaired vision is clearly disturbing her role performance as a student, as evidenced by her lower grades. Although the other options may also represent accurate diagnoses for this patient, they do not flow from the data presented.

A resident who is called to see a patient in the middle of the night is leaving the unit but then remembers that he forgot to write a new order for a pain medication a nurse had requested for another patient. Tired and already being paged to another unit, he verbally tells the nurse the order and asks the nurse to document it on the health care provider's order sheet. What is the nurse's BEST response?

a) State: "Thank you for taking care of this! I'll be happy to document the order on the health care provider's order sheet." b) Get a second nurse to listen to the order, and after writing the order on the health care provider order sheet, have both nurses sign it. c) State: "I am sorry, but VOs can only be given in an emergency situation that prevents us from writing them out. I'll bring the chart and we can do this quickly." d) Try calling another resident for the order or wait until the next shift. Ans: c. In most facilities, the only circumstance in which an attending physician, nurse practitioner, or house officer may issue orders verbally is in a medical emergency, when the physician or nurse practitioner is present but finds it impossible, due to the emergency situation, to write the order. Trying to call another resident for the order or waiting until the next shift would be inappropriate; the patient should not have to wait for the pain medication, and a resident is available who can immediately write the order.

A patient is in the late stages of AIDS, with alterations to the brain as well as other major organ systems. The patient complains of loneliness because of friends being "afraid to visit." Based on this data, what would the nurse determine to be the least likely underlying etiology for this patient's sensory problems?

a) Stimulation b) Reception c) Transmission-perception-reaction d) Emotional responses Ans: d. Emotional responses are an effect of sensory deprivation, and although they may be occurring with this patient, they are not the underlying etiology for the patient's condition. This patient is receiving decreased environmental stimuli (e.g., from lack of friends), and is more than likely experiencing problems with reception because of major organ involvement. In addition, impaired brain function will impair impulse transmission-perception-reaction.

A patient in an intensive care burn unit for 1 week is in pain much of the time and has his face and both arms heavily bandaged. His wife visits every evening for 15 minutes at 1800, 1900, and 2000. A heart monitor beeps for a patient on one side, and another patient moans frequently. Which patient assessment would the nurse make based on this data?

a) Sufficient sensory stimulation b) Deficient sensory stimulation c) Excessive sensory stimulation d) Both sensory deprivation and overload Ans: d. This patient's bandages may result in deficient sensory stimulation (sensory deprivation), and the monitors and other sounds in the intensive care burn unit may cause a sensory overload. All other options are incomplete responses.

The nursing process ensures that nurses are person centered rather than task centered. Rather than simply approaching a patient to take vital signs, the nurse thinks, "How is Mrs. Barclay today? Are our nursing actions helping her to achieve her goals? How can we better help her?" This demonstrates which characteristic of the nursing process?

a) Systematic b) Interpersonal c) Dynamic d) Universally applicable in nursing situations Ans: b. Interpersonal. All of the other options are characteristics of the nursing process, but the conversation and thinking quoted best illustrates the interpersonal dimension of the nursing process.

A student nurse is on a clinical rotation at a busy hospital unit. The RN in charge tells the student to change a surgical dressing on a patient while she takes care of other patients. The student has not changed dressings before and does not feel confident performing the procedure. What would be the student's best response?

a) Tell the RN that he or she lacks the technical competencies to change the dressing independently. b) Assemble the equipment for the procedure and follow the steps in the procedure manual. c) Ask another student nurse to work collaboratively with him or her to change the dressing. d) Report the RN to his or her instructor for delegating a task that should not be assigned to student nurses. Ans: a. Student nurses should notify their nursing instructor or nurse mentor if they believe they lack any competencies needed to safely implement the care plan. It is within the realm of a student nurse to change a dressing if he or she is technically prepared to do so.

A home health care nurse is using the steps of the SDLC, to design a new system for home health care documentation. The nurse analyzes the old system and develops plans for the new system. What is the next step of the nurse in this process?

a) Test b) Design c) Implement d) Evaluate Ans: b. The SDLC requires focus in the areas of Analyze and Plan, Design and Build, Test, Train, Implement, Maintain, and Evaluate. After analyzing and planning the new system, the nurse would move on to the design step in which the basic design of the new system is developed. The nurse would then test the system, train employees, and implement, maintain, and evaluate the new system in that order.

When a nurse enters the patient's room to begin a nursing history, the patient's wife is there. After introducing herself to the patient and his wife, what should the nurse do?

a) Thank the wife for being present. b) Ask the wife if she wants to remain. c) Ask the wife to leave. d) Ask the patient if he would like the wife to stay. Ans: d. The patient has the right to indicate whom he would like to be present for the nursing history and exam. The nurse should neither presume that he wants his wife there nor that he does not want her there. Similarly, the choice belongs to the patient, not the wife.

A nurse is caring for patients of diverse cultures in a community health care facility. Which characteristics of cultural diversity that exist in the United States should the nurse consider when planning culturally competent care? Select all that apply.

a) The United States has become less inclusive of same-sex couples. b) Cultural diversity is limited to people of varying cultures and races. c) Cultural diversity is separate and distinct from health and illness. d) People may be members of multiple cultural groups at one time. e) Culture guides what is acceptable behavior for people in a specific group. f) Cultural practices may evolve over time but mainly remain constant. Ans: d, e, f. A person may be a member of multiple cultural, ethnic, and racial groups at one time. Culture guides what is acceptable behavior for people in a specific group. Cultural practices and beliefs may evolve over time, but they mainly remain constant as long as they satisfy a group's needs. The United States has become more (not less) inclusive of same-sex couples. The definition of cultural diversity includes, but is not limited to, people of varying cultures, racial and ethnic origin, religion, language, physical size, biological sex, sexual orientation, age, disability, socioeconomic status, occupational status, and geographic location. Cultural diversity, including culture, ethnicity, and race, is an integral component of both health and illness.

An RN on a surgical unit is behind schedule administering medications. Which of the RN's other tasks can be safely delegated to a UAP?

a) The assessment of a patient who has just arrived on the unit b) Teaching a patient with newly diagnosed diabetes about foot care c) Documentation of a patient's I & O on the flow chart d) Helping a patient who has recently undergone surgery out of bed for the first time Ans: c. Documenting a patient's I & O on a flow chart may be delegated to a UAP. Professional nurses are responsible for the initial patient assessment, discharge planning, health education, care planning, triage, interpretation of patient data, care of invasive lines, administering parenteral medications. What they can delegate are assistance with basic care activities (bathing, grooming, ambulation, feeding) and things like taking vital signs, measuring intake and output, weighing, simple dressing changes, transfers, and post mortem care.

A charge nurse in a busy hospital manages a skilled nursing unit using an autocratic style of leadership. Which leadership tasks BEST represent this style of leadership? Select all that apply.

a) The charge nurse polls the other nurses for input on nursing protocols. b) The charge nurse dictates break schedules for the other nurses. c) The charge nurse schedules a mandatory in-service training on new equipment. d) The charge nurse allows the other nurses to divide up nursing tasks. e) The charge nurse delegates nursing responsibilities to the staff. f) The charge nurse encourages the nurses to work independently. Ans: b, c, e. Autocratic leadership involves the leader assuming control over the decisions and activities of the group, such as dictating schedules and work responsibilities, and scheduling mandatory in-service training. Polling other nurses is an example of democratic leadership, which is characterized by a sense of equality among the leader and other participants, with decisions and activities being shared. In laissez-faire leadership, the leader relinquishes power to the group and encourages independent activity by group members. Examples of laissez-faire leadership style are allowing the nurses to divide up the tasks and encouraging them to work independently.

A nurse who is newly hired to manage a busy pediatric office is encouraged to use a transactional leadership style when dealing with subordinates. Which activities best exemplify the use of this type of leadership? Select all that apply.

a) The manager institutes a reward program for employees who meet goals and work deadlines. b) The manager encourages the other nurses to participate in health care reform by joining nursing organizations. c) The manager promotes compliance by reminding subordinates that they have a good salary and working conditions. d) The manager makes sure all the employees are kept abreast of new developments in pediatric nursing. e) The manager works with subordinates to accomplish all the nursing tasks and goals for the day. f) The manager allows the other nurses to set their own schedules and perform nursing care as they see fit. Ans: a, c. Instituting a reward program and reminding workers that they have a good salary and working conditions are examples of transactional leadership, which is based on a task-and-reward orientation. Team members agree to a satisfactory salary and working conditions in exchange for commitment and compliance to their leader. Encouraging nurses to participate in health care reform is an example of a transformational leadership style. Ensuring that employees keep abreast of new developments in nursing care is a characteristic of quantum leadership. The group and leader work together to accomplish mutually set goals and outcomes with the democratic leadership style, and the laissez-faire style encourages independent activity by group members, such as setting their own schedules and work activities.

A nurse is using time management techniques when planning activities for patients. Which nursing action reflects effective time management?

a) The nurse asks patients to prioritize what they want to accomplish each day b) The nurse includes a "nice to do" for every "need to do" task on the list c) The nurse "front loads" the schedule with "must do" priorities d) The nurse avoids helping other nurses if scheduling does not permit it Ans: a. By asking the patient to prioritize what they want to accomplish each day, the nurse is demonstrating an effective time management technique. In order to manage time, the nurse should establish goals and priorities for each day, differentiating "need to do" from "nice to do" tasks; the nurse should include the patient in this process. The nurse should also establish a time line, allocating priorities to hours in the workday in order to keep track of falling behind and correct the problem before the day is lost. The nurse should use teamwork appropriately to enhance the schedule.

A nurse is assessing an older adult patient for kinesthetic and visceral disturbances. Which techniques would the nurse use for this assessment? Select all that apply.

a) The nurse asks the patient if he is bored, and if so, why. b) The nurse asks the patient if anything interferes with the functioning of his senses. c) The nurse asks the patient if he noticed any changes in the way he perceives his body. d) The nurse asks the patient if he has found it difficult to communicate verbally. e) The nurse notes if the patient withdraws from being touched. f) The nurse notes if the patient seems unsure of his body parts or position. Ans: c, e, f. To assess for kinesthetic and visceral disturbances, the nurse would assess for perceived body changes inside and out, and changes in body parts or position. Asking if the patient is bored assesses stimulation. Asking if anything interferes with his senses assesses reception. Asking about difficulty communicating assesses for transmission-perception-reaction.

A nurse is using the implementation step of the nursing process to provide care for patients in a busy hospital setting. Which nursing actions best represent this step? Select all that apply.

a) The nurse carefully removes the bandages from a burn victim's arm. b) The nurse assesses a patient to check nutritional status. c) The nurse formulates a nursing diagnosis for a patient with epilepsy. d) The nurse turns a patient in bed every 2 hours to prevent pressure injuries. e) The nurse checks a patient's insurance coverage at the initial interview. f) The nurse checks for community resources for a patient with dementia. Ans: a, d, f. During the implementing step of the nursing process, nursing actions planned in the previous step are carried out. The purpose of implementation is to assist the patient in achieving valued health outcomes: promote health, prevent disease and illness, restore health, and facilitate coping with altered functioning. Assessing a patient for nutritional status or insurance coverage occurs in the assessment step, and formulating nursing diagnoses occurs in the diagnosing step.

After instituting a new system for recording patient data, a nurse evaluates the "usability" of the system. Which actions by the nurse BEST reflect this goal? Select all that apply.

a) The nurse checks that the screens are formatted to allow for ease of data entry. b) The nurse reorders the screen sequencing to maximize effective use of the system. c) The nurse ensures that the computers can be used by specified users effectively. d) The nurse checks that the system is intuitive, and supportive of nurses. e) The nurse improves end-user skills and satisfaction with the new system. f) The nurse ensures patient data is able to be shared across health care systems. Ans: a, c, d. Usability refers to the extent to which a product can be used by specified users to achieve specified goals with effectiveness, efficiency, and satisfaction in a specified context of use. Checking that screens are formatted to allow ease of data entry, ensuring that computers can be used by specified users effectively, and checking that the system is intuitive and supportive of nurses are all tasks related to the "usability" of the system. Reordering screen sequencing to maximize use and improving end-user skills and satisfaction with the new system refers to optimization. The ability to share patient data across health care systems is termed interoperability.

A nurse is collecting evaluative data for a patient who is finished receiving chemotherapy for an osteosarcoma. Which nursing action represents this step of the nursing process?

a) The nurse collects data to identify health problems. b) The nurse collects data to identify patient strengths. c) The nurse collects data to justify terminating the care plan. d) The nurse collects data to measure outcome achievement. Ans: d. The nurse collects evaluative data to measure outcome achievement. While this may justify terminating the care plan, that is not necessarily so. Data to assess health problems and patient variables are collected during the first step of the nursing process.

A nurse is using the steps in informatics evaluation to evaluate the use of a portal as a patient resource. What are examples of activities that might occur in the "determining the question" step? Select all that apply.

a) The nurse develops a clear, focused question to determine the data to be collected. b) The nurse determines what to evaluate.The nurse determines how the data ultimately should be reported. c) The nurse decides what specific data elements need to be collected. d) The nurse clarifies exactly how the data will be collected. e) The nurse performs comprehensive documentation of the data collected. Ans: a, c. The nurse develops a clear, focused question to determine the data to be collected and the nurse determines how the data ultimately should be reported during the "determine the question" step. The nurse determines what to evaluate during the step "determine what will be evaluated." The nurse decides what specific data elements need to be collected during the "determine the needed data" step. The nurse clarifies exactly how the data will be collected during the "determine the data collection method and sample size" step. The nurse performs comprehensive documentation of the data collected during the "document your outcome evaluation" step.

A nurse manager is attempting to update a health care provider's office from paper to electronic health records (EHR) by using the eight-step process for planned change. Place the following actions in the order in which they should be initiated:

a) The nurse devises a plan to switch to EHR. b) The nurse records the time spent on written records versus EHR. c) The nurse attains approval from management for new computers. d) The nurse analyzes all options for converting to EHR. e) The nurse installs new computers and provides an in-service for the staff. f) The nurse explores possible barriers to changing to EHR. g)The nurse follows up with the staff to check compliance with the new system. h) The nurse evaluates the effects of changing to EHR. Ans: b, f, d, c, a, e, h, g. Planned change involves the following steps: (1) recognize symptoms that indicate a change is needed and collect data, (2) identify a problem to be solved through change, (3) determine and analyze alternative solutions, (4) select a course of action from possible solutions, (5) plan for making the change, (6) implement the change, (7) evaluate the change, and (8) stabilize the change.

A nurse is planning care for a patient who was admitted to the hospital for treatment of a drug overdose. Which nursing actions are related to the outcome identification and planning step of the nursing process? Select all that apply.

a) The nurse formulates nursing diagnoses. b) The nurse identifies expected patient outcomes. c) The nurse selects evidence-based nursing interventions. d) The nurse explains the nursing care plan to the patient. e) The nurse assesses the patient's mental status. f) The nurse evaluates the patient's outcome achievement. Ans: b, c, d. During the outcome identification and planning step of the nursing process, the nurse works in partnership with the patient and family to establish priorities, identify and write expected patient outcomes, select evidence-based nursing interventions, and communicate the nursing care plan. Although all these steps may overlap, formulating and validating nursing diagnoses occur most frequently during the diagnosing step of the nursing process. Assessing mental status is part of the assessment step, and evaluating patient outcomes occurs during the evaluation step of the nursing process.

A female patient who is receiving chemotherapy for breast cancer tells the nurse, "The treatment for this cancer is worse than the disease itself. I'm not going to come for my therapy anymore." The nurse responds by using critical thinking skills to address this patient problem. Which action is the first step the nurse would take in this process?

a) The nurse judges whether the patient database is adequate to address the problem. b) The nurse considers whether or not to suggest a counseling session for the patient. c) The nurse reassesses the patient and decides how best to intervene in her care. d) The nurse identifies several options for intervening in the patient's care and critiques the merit of each option. Ans: c. The first step when thinking critically about a situation is to identify the purpose or goal of your thinking. Reassessing the patient helps to discipline thinking by directing all thoughts toward the goal. Once the problem is addressed, it is important for the nurse to judge the adequacy of the knowledge, identify potential problems, use helpful resources, and critique the decision.

A nurse is asked to act as a mentor to a new nurse. Which nursing action is related to this process?

a) The nurse mentor accepts payment to introduce the new nurse to his or her responsibilities b) The nurse mentor hires the new nurse and assigns duties related to the position c) The nurse mentor makes it possible for the new nurse to participate in professional organizations d) The nurse mentor advises and assists the new nurse to adjust to the work environment of a busy emergency department Ans: d. Mentorship is a relationship in which an experienced person (the mentor) advises and assists a less experienced person (protégé). This is an effective way of easing a new nurse into leadership responsibilities. An experienced nurse who is paid to introduce an employee to new responsibilities through teaching and guidance describes a preceptor, not a mentor. The nurse mentor does not hire or schedule new nurses. Nurses do not need mentors to join professional organizations.

A nurse is a servant leader working in an economically depressed community to set up a free mobile health clinic for the residents. Which actions by the leader BEST exemplify a key practice of servant leaders? Select all that apply

a) The nurse motivates coworkers to solicit funding to set up the clinic. b) The nurse sets only realistic goals that are present oriented and easily achieved. c) The nurse forms an autocratic governing body to keep the project on track. d) The nurse spends time with supporters to help them grow in their roles. e) The nurse first ensures that other's lowest priority needs are served. f) The nurse prizes leadership because of the need to serve others. Ans: a, d, f. In order to serve as servant leaders, nurses need to invest in those who support the organization's values, show passion, can play to their strengths, and demonstrate a positive attitude. They should develop their vision to see the future related to a current anticipated need, and motivate others to follow and engage. They also need to provide ongoing opportunities for collaborations, sharing, reflection, encouragement, and celebration, as well as hard work. The servant leader allows others to have a voice, to exercise control, and to practice leading themselves. The servant first makes sure that other people's highest priority needs are being served. The best test, and most difficult to administer, is: Do those served grow as people? Do they, while being served, become healthier, wiser, freer, more autonomous, more likely themselves to become servants?

A nurse is using critical pathway methodology for choosing interventions for a patient who is receiving chemotherapy for breast cancer. Which nursing actions are characteristics of this system being used when planning care? Select all that apply.

a) The nurse uses a minimal practice standard and is able to alter care to meet the patient's individual needs. b) The nurse uses a binary decision tree for stepwise assessment and intervention. c) The nurse is able to measure the cause-and-effect relationship between pathway and patient outcomes. d) The nurse uses broad, research-based practice recommendations that may or may not have been tested in clinical practice. e) The nurse uses preprinted provider orders used to expedite the order process after a practice standard has been validated through research. f) The nurse uses a decision tree that provides intense specificity and no provider flexibility. Ans: a, c. A critical pathway represents a sequential, interdisciplinary, minimal practice standard for a specific patient population that provides flexibility to alter care to meet individualized patient needs. It also offers the ability to measure a cause-and-effect relationship between pathway and patient outcomes. An algorithm is a binary decision tree that guides stepwise assessment and intervention with intense specificity and no provider flexibility. Guidelines are broad, research-based practice recommendations that may or may not have been tested in clinical practice, and an order set is a preprinted provider order used to expedite the order process after a practice standard has been validated through analytical research.

The nurse uses blended competencies when caring for patients in a rehabilitation facility. Which examples of interventions involve cognitive skills? Select all that apply.

a) The nurse uses critical thinking skills to plan care for a patient. b) The nurse correctly administers IV saline to a patient who is dehydrated. c) The nurse assists a patient to fill out an informed consent form. d) The nurse learns the correct dosages for patient pain medications. e) The nurse comforts a mother whose baby was born with Down syndrome. f) The nurse uses the proper procedure to catheterize a female patient. Ans: a, d. Using critical thinking and learning medication dosages are cognitive competencies. Performing procedures correctly is a technical skill, helping a patient with an informed consent form is a legal/ethical issue, and comforting a patient is an interpersonal skill.

A registered nurse is writing a diagnosis for a patient who is in traction because of multiple fractures from a motor vehicle accident. Which nursing actions are related to this step in the nursing process? Select all that apply.

a) The nurse uses the nursing interview to collect patient data. b) The nurse analyzes data collected in the nursing assessment. c) The nurse develops a care plan for the patient. d) The nurse points out the patient's strengths. e) The nurse assesses the patient's mental status. f) The nurse identifies community resources to help his family cope. Ans: b, d, f. The purposes of diagnosing are to identify how an individual, group, or community responds to actual or potential health and life processes; identify factors that contribute to or cause health problems (etiologies); and identify resources or strengths the individual, group, or community can draw on to prevent or resolve problems. In the diagnosing step of the nursing process, the nurse interprets and analyzes data gathered from the nursing assessment, identifies patient strengths, and identifies resources the patient can use to resolve problems. The nurse assesses and collects patient data in the assessment step and develops a care plan in the planning step of the nursing process.

Population health addresses the health status and health issues of aggregate populations and addresses ways in which resources may be allocated to address these concerns. What is the driving force behind the use by health corporations of analytics and big data to support population health?

a) The transition from fee-for-service models to value-based payment models b) A growing older population with more complicated health needs c) The overcrowding and understaffing of hospitals d) The shortage of health care professionals, particularly nurses Ans: a. Information technology is a part of the core infrastructure on which population health can be assessed and addressed. As organizations transition from the traditional fee-for-service model to value-based payment models (including ACOs), data, information, and knowledge about populations rather than individual patients will be required. A growing older population with more complicated health needs, the overcrowding and understaffing of hospitals, and the shortage of health care professionals, particularly nurses, may be affected by population health assessment, but are not the driving force for the development of this technology.

A patient's spinal cord was severed, causing paralysis from the waist down. When obtaining data about this patient, which component of the sensory experience would be a priority for the nurse to assess?

a) Transmission of tactile stimuli b) Adequate stimulation in the environment c) Reception of visual and auditory stimuli d) General orientation and ability to follow commands Ans: a. Below-the-waist paralysis makes the transmission of tactile stimuli a problem. Although the other options may be assessed, they are indirectly related to his paralysis and of less importance at this time.

A nurse working in a long-term care facility bases patient care on five caring processes: knowing, being with, doing for, enabling, and maintaining belief. This approach to patient care best describes whose theory?

a) Travelbee's b) Watson's c) Benner's d) Swanson's Ans: d. Swanson (1991) identifies five caring processes and defines caring as "a nurturing way of relating to a valued other toward whom one feels a personal sense of commitment and responsibility." Travelbee (1971), an early nurse theorist, developed the Human-to-Human Relationship Model, and defined nursing as an interpersonal process whereby the professional nurse practitioner assists an individual, family, or community to prevent or cope with the experience of illness and suffering, and if necessary to find meaning in these experiences. Benner and Wrubel (1989) wrote that caring is a basic way of being in the world, and that caring is central to human expertise, curing, and healing. Watson's theory is based on the belief that all humans are to be valued, cared for, respected, nurtured, understood, and assisted.

A nurse is caring for a man with a severe hearing deficit who is able to read lips and use sign language. Which nursing intervention would best prevent sensory alterations for this patient?

a) Turn the radio or television volume up very loud and close the door to his room. b) Prevent embarrassment and emotional discomfort as much as possible. c) Provide daily opportunity for him to participate in a social hour with 6 to 8 people. d) Encourage daily participation in exercise and physical activity. Ans: c. Although all the options listed are appropriate, providing daily opportunities for this patient to participate in a social hour builds on his strength of being able to lip-read and provides sufficient sensory stimulation to prevent sensory deprivation resulting from his hearing loss, thereby meeting his needs.

A nurse is testing a new computer program designed to store patient data. In what phase of testing would the nurse determine if the system can handle high volumes of end-users or care providers using the system at the same time?

a) Unit b) Function c) Integration d) Performance Ans: d. Performance testing is more technical and ensures proper functioning of the system when there are high volumes of end-users or care providers using the system at the same time, ensuring it can handle the load. Unit testing is basic testing that occurs initially. Function testing uses test scripts to validate that a system is working as designed for one particular function. Integration testing uses test script to validate that a system is working as designed for an entire workflow that integrates multiple components of the system.

Nurses test new technology in phases. In which phase would the nurse "test drive" the new system?

a) Unit b) Function c) User acceptance d) Integration Ans: c. During the phase "user acceptance," the nurse would "test drive" the new system to ensure it's working as designed. Unit testing is basic testing that occurs initially. Function testing uses test scripts to validate that a system is working as designed for one particular function. Integration testing uses test script to validate that a system is working as designed for an entire workflow that integrates multiple components of the system.

A nurse is assessing a patient in a long-term care facility. The nurse notes that the patient is at risk for sensory deprivation due to limited activity related to severe rheumatoid arthritis. Which interventions would the nurse recommend based on this finding? Select all that apply.

a) Use a lower tone when communicating with the patient. b) Provide interaction with children and pets. c) Decrease environmental noise. d) Ensure that the patient shares meals with other patients. e) Discourage the use of sedatives. f) Provide adequate lighting and clear pathways of clutter. Ans: b, d, e. For a patient who has sensory deprivation, the nurse should provide interaction with children and pets, ensure that the patient shares meals with other patients, and discourage the use of sedatives. Using a lower tone of voice is appropriate for a patient who has a hearing deficit. Decreasing environmental noise is an intervention for sensory overload. Providing adequate lighting and removing clutter is an intervention for a vision deficit.

A young Hispanic mother comes to the local clinic because her baby is sick. She speaks only Spanish and the nurse speaks only English. What is the appropriate nursing intervention?

a) Use short words and talk more loudly. b) Ask an interpreter for help. c) Explain why care can't be provided. d) Provide instructions in writing. Ans: b. The nurse should ask an interpreter for help. Many facilities have a qualified interpreter who understands the health care system and can reliably provide assistance. Using short words, talking loudly, and providing instructions in writing will not help the nurse communicate with this patient. Explaining why care can't be provided is not an acceptable choice because the nurse is required to provide care; also, since the patient doesn't speak English, she won't understand what the nurse is saying.

A nurse designing a new EHR system for a pediatric office follows usability concepts in system design. Which concepts are recommended in system design? Select all that apply.

a) Users should not explore with forgiveness for unintended consequences. b) Shortcuts for frequent users should not be incorporated into the system. c) Content emphasis should be on information needed for decision making. d) The less times users need to apply prior experience to a new system the better. e) All the information needed should be presented to reduce cognitive load. f) The number of steps it takes to complete tasks should be minimized. Ans: c, e, f. When designing a system, content emphasis should be on information needed for decision making. All the information needed should be presented to reduce cognitive load. The number of steps it takes to complete tasks should be minimized. The more users can apply prior experience to a new system, the lower the learning curve, the more effective their usage, and the fewer their errors. Forgiveness means that a design allows the user to discover it through exploration without fear of disastrous results. This approach accelerates learning while building in protections against unintended consequences. One of the most direct ways to facilitate efficient user interactions is to minimize the number of steps it takes to complete tasks and to provide shortcuts for use by frequent and/or experienced users.

A nurse is identifying outcomes for a patient who has a leg ulcer related to diabetes. What is an example of an affective outcome for this patient?

a) Within 1 day after teaching, the patient will list three benefits of continuing to apply moist compresses to leg ulcer after discharge. b) By 6/12/20, the patient will correctly demonstrate application of wet-to-dry dressing on leg ulcer. c) By 6/19/20, the patient's ulcer will begin to show signs of healing (e.g., size shrinks from 3 to 2.5 in). d) By 6/12/20, the patient will verbalize valuing health sufficiently to practice new health behaviors to prevent recurrence of leg ulcer. Ans: d. Affective outcomes describe changes in patient values, beliefs, and attitudes. Cognitive outcomes (a) describe increases in patient knowledge or intellectual behaviors; psychomotor outcomes (b) describe the patient's achievement of new skills; and (c) is an outcome describing a physical change in the patient.


संबंधित स्टडी सेट्स

Ch. 21: The Child with Respiratory Dysfunction

View Set

Intro to Surgical Technology Final Part 2

View Set

UIUC CS 498 Data Visualization Midterm

View Set

Hand Anatomy: Intrinsic and Extrinsic

View Set

F4 M8: Goodwill, Including Impairment

View Set